You are on page 1of 37

VISIONIAS

www.visionias.in

ANSWERS & EXPLANATION


GENERAL STUDIES (P) TEST – 2965 (2020)

Q 1.C
o As per the Plate tectonic theory, the surface of the earth and the interior are not static and motionless but
are dynamic. The mobile rock beneath the rigid plates is believed to be moving in a circular manner.
o The heated material rises to the surface, spreads and begins to cool, and then sinks back into deeper
depths. This cycle is repeated over and over to generate what scientists call a convection cell or
convective flow. Heat within the earth comes from two main sources: radioactive decay and residual
heat.
o The radioactive decay of naturally occurring chemical elements - most notably uranium, thorium, and
potassium - releases energy in the form of heat, which slowly migrates toward the Earth's surface.
Residual heat is gravitational energy left over from the formation of the Earth - 4.6 billion years ago - by
the "falling together" and compression of cosmic debris.

Q 2.B
o The black soils are generally clayey, deep and impermeable. They swell and become sticky when wet and
shrink when dried. So, during the dry season, these soils develop wide cracks. Thus, there occurs a
kind of ‗self ploughing‘. Because of this character of slow absorption and loss of moisture, the black soil
retains the moisture for a very long time, which helps the crops, especially, the rain-fed ones, to sustain
even during the dry season.

Q 3.C
o Chandoli National Park is located in the Sangli, Satara, Kolhapur, and Ratnagiri districts of the state
of Maharashtra. The park spreads along the crest of the Sahyadri Range of the northern Western Ghats.
It forms and protects many perennial water channels, water holes and the Vasant Sagar Reservoir.
Sahyadri Tiger Reserve, including all of Chandoli National Park and Koyna Wildlife Sanctuary was
declared by The National Tiger Conservation Authority as a Project Tiger, Tiger Reserve. The park
receives its water supply from the Warna River and reservoir as well as several other small streams and
rivers.
o Bengaluru Bannerghatta National Park is located about 22 Kms south of Bengaluru city of the state
of Karnataka . It is one among the few places in the world where wilderness is preserved so close to a
big city.The vision of the Bengaluru Bannerghatta National Park is to create an opportunity to the
citizens - specially to children of Bangalore Metropolitan City to have a Biological Recreation Center
very close to the city in the midst of the forests in the valley of the famous Champakadhama hills inside
the Bannerghatta National Park. To accomplish this vision, a mini zoo was created in 1972' which
gradually grown into the present National Park by 2002.
o Kudremukh National Park is a beautiful place, located in the Dakshina Kannada, Udupi and
Chikmagalur districts of the state of Karnataka. Nestled in the Western Ghats.Three important rivers,
the Tunga, the Bhadra, and the Netravati are said to have their origin here. The Tunga River and Bhadra
River flow freely through the parklands. The animals found there include malabar civets, wild dogs, sloth
bears and spotted deer.
o Nameri National Park is located in the foothills of the Eastern Himalayas in the Sonitpur District
of Assam, India, about 40 kilometres from Tezpur.Nameri is a birder's paradise with over 300 species of
birds.The area is criss-crossed by the river Jia- Bhoroli and its tributaries namely the Diji, Dinai,
Doigurung, Nameri, Dikorai, Khari etc. There is a good prey base in the form of Sambar, Barking deer,
Hog Deer, Wild Boar and Gaur. About 3000 domestic cattle also form part of this prey base for Tiger and
Leopards.

1 www.visionias.in ©Vision IAS


Q 4.D
o The flow of water through well-defined channels is known as ‗drainage‘ and the network of such channels
is called a ‗drainage system‘.
o The drainage pattern of an area is the outcome of the geological time period, nature and structure of rocks,
topography, slope, amount of water flowing and the periodicity of the flow. Hence, all the options are
correct.
o Some of the different types of drainage patterns are:
 Dendritic: The drainage pattern resembling the branches of a tree is known as Dendritic. It is formed
in areas of gently sloping land. Himalayan rivers follow Dendritic pattern
 Radial: When the rivers originate from a hill and flow in all directions, the drainage pattern is known
as Radial. The rivers originating from the Amarkantak range present a good example of it.
 Trellis: When the primary tributaries of rivers flow parallel to each other and secondary tributaries
join them at right angles, the pattern is known as Trellis. Rivers in the upper part of the Himalayan
region form a trellis pattern
 Centripetal: When the rivers discharge their waters from all directions in a lake or depression, the
pattern is known as Centripetal pattern. For example, Loktak lake in Manipur

Q 5.C
o There are two rain-bearing systems in India.
 First originates in the Bay of Bengal causing rainfall over the plains of north India.
 Second is the Arabian Sea current of the southwest monsoon which brings rain to the west coast of
India.
o The frequency of the tropical depressions originating from the Bay of Bengal varies from year to year.
Their paths over India are mainly determined by the position of ITCZ which is generally termed as
the monsoon trough. Monsoon Trough is a portion of the ITCZ and is somewhat like a convergence zone
between the wind patterns of the southern and northern hemispheres.

o As the axis of the monsoon trough oscillates, there are fluctuations in the track and direction of
these depressions, and the intensity and the amount of rainfall vary from year to year. Hence
statement 1 is correct.
o The rain which comes in spells displays a declining trend from west to east over the west coast, and from
the southeast towards the northwest over the North Indian Plain and the northern part of the
Peninsula. Hence statement 2 is correct.

2 www.visionias.in ©Vision IAS


Q 6.C
o Statement 1 is correct: When the earth‘s crust bends folding occurs, but when it cracks, faulting takes
place. Block mountains are created when large areas or blocks of earth are broken and displaced
vertically.
o Fault-block mountains are formed by the movement of large crustal blocks when forces in the Earth's
crust pull it apart (Tension), bedrock is split into two parts by tectonic activity. The faulted edges are
very steep.

o Block Mountains are formed when two tectonic plates move away from each other causing cracks on
the surface of the Earth. When parallel cracks or faults occur, the strip of land or the block of land
between them may be raised resulting in the formation of block mountains.Examples, Black forest and
the Vosges of Rhineland.
o Block mountains are also formed when the crust of the Earth sinks (Compression) on both sides of
two parallel faults. Therefore, a block mountain can be found between two rift valleys. The land
which sinks is known as graben. Examples, East African rift valleys.
o Statement 2 is not correct: The Vosges and the Black Forest of the Rhineland, Great African Rift Valley
system are the best examples.Mt. Monadnock in U.S.A is an example of a residual mountain.
o Block mountains are also called fault block mountains since they are formed due to faulting as a
result of tensile and compressive forces. Block mountains are surrounded by faults on either side of rift
valleys or grabens. The uplifted blocks are termed as horsts and the lowered blocks are called graben.

Q 7.B
o Temperate Continental (Steppe) Climate
o A steppe is a dry, grassy plain. Steppes occur in temperate climates, which lie between the tropics and
polar regions.
o Temperate regions have distinct seasonal temperature changes, with cold winters and warm summers.
o Though they lie in the Westerly wind belt, they are so remote from the maritime influence that the
grasslands are practically treeless.

3 www.visionias.in ©Vision IAS


o In the northern hemisphere, the grasslands are far more extensive and are entirely continental. Hence
statement 1 is correct.
 In Eurasia, they are called the Steppes, and stretch eastwards from the shores of the Black Sea across
the great Russian plain to the foothills of the Altai Mountains, a distance of well over 2,000 miles.
o In the southern hemisphere, there is a maritime influence in the steppe type of climate.
 Its annual precipitation is always more than the average 20 inches because of the warm ocean currents
that wash the shores of the steppe-lands here.
 Pretoria, in South Africa, has an annual precipitation of 26 inches with the wettest months in
November, December, January and February, the summer season of the southern hemisphere. There
are three months (June, July and August) without any rain. This is the period of drought. The dry
season is particularly pronounced in temperate grasslands adjoining deserts, for example in
Australia. Hence statement 2 is not correct.
o Chinook is a warm, dry wind descending the eastern slopes of the Rocky Mountains in the USA,
primarily in winter. Hence statement 3 is not correct.
o Winds of the same kind occur in other parts of the world and are known generally as foehns.
 It comes in a south-westerly direction to the Prairies and has a considerable effect on the local
pastures. It melts the snow-covered pastures and animals can be driven out of doors to graze in the
open fields.
 It actually comes with the depressions in winter or early spring from the Pacific coast ascending the
Rockies and then descending to the Prairies.

Q 8.A
o Bars, Barriers and Spits are depositional landforms formed by the action of waves and currents.
o A ridge of sand and shingle formed in the sea in the off-shore zone (from the position of low tide
waterline to seaward) lying approximately parallel to the coast is called an off-shore bar.
o An off-shore bar which is exposed due to further addition of sand is termed a barrier bar. The off-shore
bars and barriers commonly form across the mouth of a river or at the entrance of a bay. Sometimes such
barrier bars get keyed up to one end of the bay when they are called spits. Spits may also develop attached
to headlands/hills.
o The barriers, bars and spits at the mouth of the bay gradually extend leaving only a small opening of the
bay into the sea and the bay will eventually develop into a lagoon. The lagoons get filled up gradually by
sediment coming from the land or from the beach itself (aided by wind) and a broad and wide coastal
plain may develop replacing a lagoon.

Q 9.D
o Statements 1 and 2 are correct : Two major island groups in India are present in the Bay of Bengal and
the other in the Arabian Sea. The Bay of Bengal island groups consist of about 572 islands/islets. These
are situated roughly between 6°N-14°N and 92°E -94°E. The two principal groups of islets include the
Ritchie‘s archipelago and the Labyrinth island.

4 www.visionias.in ©Vision IAS


o Among these, the Andamans are situated in the north and the Nicobar in the south. Both Andaman and
Nicobar are separated by a waterbody which is called the Ten degree channel. These islands are an
elevated portion of submarine mountains.
o Statement 3 is correct: Some smaller islands are volcanic in origin. Barren island, the only active
volcano in India is situated in the Andaman sea. It is historically active volcano along the N-S-trending
volcanic arc extending between Sumatra and Burma.
o Other Related Information:
 Once a hill range extending from Myanmar to Indonesia, these picturesque undulating islands are
covered with dense rain-fed, damp and evergreen forests and endless varieties of exotic flora and
fauna.
 Most of these islands (about 550) are in the Andaman Group, 28 of which are inhabited. The smaller
Nicobars, comprise some 22 main islands (10 inhabited).
 These islands also boast of freedom fighting days‘ historically significant landmarks viz. Cellular
Jail, Netaji Subhas Chandra Bose Island, Viper Island, Hopetown and Mount Harriet.
 The Andaman and Nicobar Islands have been declared as two of the 218 endemic bird area of the
world. The Andaman Wood Pigeon, Andaman Padauk and Dugong are declared as State Bird,
State Tree and State Animal respectively.
 TThese islands are blessed with the bounties of both south-west and north-east monsoons.
 Indigenous people of Andamans were: the Great Andamanese, who collectively represented at
least 10 distinct sub groups and languages; the Jarawa: the jungle (or Rutland Jarawa); the Onge;
and the Sentinelese (the most isolated of all the groups).
 The indigenous peoples of the Nicobars include two main groups: the Nicobarese, or
Nicobari living throughout many of the islands; and the Shompen, restricted to the interior of Great
Nicobar.
 Port Blair is the capital of the Union Territory of Andaman & Nicobar Islands.

Q 10.B
o A characteristic feature of rainfall in India is its variability. The variability of rainfall is computed with
the help of the following formula:
C.V. = Standard Deviation x 100/ Mean; where C.V. is the coefficient of variation.
o The values of coefficient of variation show the change from the mean values of rainfall. The actual
rainfall in some places deviates from 20-50 per cent. The values of coefficient of variation show
variability of rainfall in India.
o A variability of less than 25 per cent exists on the western coasts, Western Ghats,
northeastern Peninsula, eastern plains of the Ganga, northeastern India, Uttarakhand and Himachal
Pradesh and south-western part of Jammu and Kashmir. These areas have an annual rainfall of over 100
cm.
o A variability of over 50 per cent exists in the western part of Rajasthan, northern part of Jammu and
Kashmir and interior parts of the Deccan plateau. These areas have an annual rainfall of less than 50 cm.
o Rest of India (including Punjab plains) has a variability of 25-50 per cent and these areas receive an
annual rainfall between 50 -100 cm. Hence the correct answer is option (b).

Q 11.B
o The retreating southwest monsoon season is marked by clear skies and rise in temperature. The land is
still moist. Owing to the conditions of high temperature and humidity, the weather becomes rather
oppressive. This is commonly known as the ‗October heat‘. Hence option (b) is the correct answer.
o In the second half of October, the mercury begins to fall rapidly, particularly in northern India. The
weather in the retreating monsoon is dry in north India but it is associated with rain in the eastern part of
the Peninsula. Here, October and November are the rainiest months of the year.

Q 12.B
o Hydrological Drought: It results when the availability of water in different storages and reservoirs
like aquifers, lakes, reservoirs, etc. falls below what the precipitation can replenish. Hence option
(b) is the correct answer.
o Meteorological Drought: It is a situation when there is a prolonged period of inadequate rainfall marked
with mal-distribution of the same over time and space.
o Agricultural Drought: It is also known as soil moisture drought, characterised by low soil moisture that
is necessary to support the crops, thereby resulting in crop failures. Moreover, if an area has more than 30
per cent of its gross cropped area under irrigation, the area is excluded from the drought-prone category.
5 www.visionias.in ©Vision IAS
o Ecological Drought: When the productivity of a natural ecosystem fails due to the shortage of water and
as a consequence of ecological distress, damages are induced in the ecosystem.

Q 13.A
o Geological time is an important metric for analysis events. The period is the basic unit of geological time
in which a single type of rock system is formed. Two or more periods comprise a geological Era. Two or
more Eras form an Eon, the largest division of geologic time. Some periods are divided into epochs.
o The descending sequence from Eon to Age is - Eon > Era > Period > Epoch > Age. Hence option (a)
is the correct answer.
o Recently, three more ages have been added by the scientists to the Holocene epoch. The Holocene epoch
commenced 11,700 years ago after the end of the last ice age. Since that time, the earth‘s climate has
continued to fluctuate.
o First, there was a warm period that lasted from 11,700 to about 8,300 years ago. Scientists have named
this age the Greenlandian age. Next, the Earth went through a gradual cooling period from about 8,300 to
4,200 years ago, and this is now known as the Northgrippian age. The last age of the Holocene began
4,200 years ago during a worldwide megadrought, and it has been named the Meghalayan age.

Q 14.A
o Dissected plateaus are formed as a consequence of the continual process of weathering and erosion by
running water and winds where high and extensive plateaus are gradually worn down, and their surfaces
made irregular.
o In drier countries, vertical corrasion by rivers and abrasion by winds will dissect the plateau into steep-
sided tabular masses which are termed as Mesas and Buttes, intersected by deep canyons. Buttes were
once part of flat, elevated areas of land known as mesas or plateaus. In fact, the only difference between a
mesa and a butte is its size. Most geographers say a butte is taller than it is wide, while a mesa is a much
larger, slightly less elevated feature. This is a common feature of arid and semi-arid areas. For e.g. In the
South-Western USA. Hence, option (a) is the correct answer.

6 www.visionias.in ©Vision IAS


Q 15.A
o There are three main reasons for the excessive cold in north India during the winter season :
 Continentality- States like Punjab, Haryana and Rajasthan being far away from the moderating
influence of sea experience continental climate.
 The snowfall in the nearby Himalayan ranges creates a cold wave situation.
 Around February, the cold winds coming from the Caspian Sea and Turkmenistan bring cold wave
along with frost and fog over the northwestern parts of India.
 The easterly jet stream sets in along 15°N latitude only after the western jet stream has withdrawn
itself from the region i.e around the month of June. This easterly jet stream is held responsible for
the burst of the monsoon in India. It does not play a role in an excessive cold climate in North India
during the winter season.
o Hence, all the options are correct.

Q 16.B
o Cumulus: This is a vertical cloud with a rounded top and horizontal base, typical of humid tropical
regions, associated with up-rising convectional currents. Its great white globular masses may look grey
against the sun but it is a fair-weather cloud. Hence, option (b) is correct.
o Cirrus: This looks fibrous and appears like wisps in the blue sky; it is often called 'mares' tails'. It
indicates fair weather and often gives a brilliant sunset.
o Nimbostratus: This is a dark, dull cloud, clearly layered, and is also known as a 'rain cloud', It brings
continuous rain, snow or sleet.
o Cumulonimbus: This is, in fact, an overgrown cumulus cloud, extending for a tremendous vertical height
from a base of 2,000 feet to over 30,000 feet. Its black and white globular masses take a fantastic range of
shapes. Its cauliflower top often spreads out like an anvil. This is frequently seen in tropical afternoons. It
is also referred to as a 'thunder-cloud' and brings convectional rain, accompanied by lightning and
thunder.

7 www.visionias.in ©Vision IAS


Q 17.C
o Statement 1 is not correct: Igneous rocks form out of magma and lava from the interior of the earth and
known as primary rocks. The igneous rocks are formed when magma cools and solidifies. When magma
in its upward movement cools and turns into the solid form it is called igneous rock. The process of
cooling and solidification can happen in the earth‘s crust or on the surface of the earth.
o Statement 2 is correct: Igneous rocks are classified based on texture. The texture depends upon the size
and arrangement of grains or other physical conditions of the materials. If molten material is cooled
slowly at great depths, mineral grains may be very large. Sudden cooling (at the surface) results in small
and smooth grains. Intermediate conditions of cooling would result in intermediate sizes of grains making
up igneous rocks.
o Statement 3 is correct: Granite, gabbro, pegmatite, basalt, volcanic breccia and tuff are some of the
examples of igneous rocks.

Q 18.C
o In our country, debris avalanches and landslides occur very frequently in the Himalayas. There are many
reasons for this. One, the Himalayas are tectonically active. They are mostly made up of sedimentary
rocks and unconsolidated and semi-consolidated deposits. The slopes are very steep.
o Compared to the Himalayas, the Nilgiris bordering Tamil Nadu, Karnataka, Kerala and the
Western Ghats along the west coast are relatively tectonically stable and are mostly made up of very
hard rocks; but, still, debris avalanches and landslides occur though not as frequently as in the
Himalayas, in these hills because.
 Many slopes are steeper with almost vertical cliffs and escarpments in the Western Ghats and Nilgiris.
 Mechanical weathering due to temperature changes and ranges is pronounced.
 They receive heavy amounts of rainfall over short periods.
o So, there is almost direct rock fall quite frequently in these places along with landslides and debris
avalanches.

Q 19.C
o Comparison between the Himalayan and the Peninsular River:
 Himalayan rivers originate from the lofty Himalayan ranges while Peninsular Rivers originate in the
Peninsular Plateau.
 Himalayan rivers have large basins and catchment areas while Peninsular rivers have small basins and
catchment areas.
 Himalayan rivers flow through deep V – shaped valleys called gorges. These gorges have been carved
out by down cutting carried on side by side with the uplift of the Himalayas. While, the Peninsular
rivers flow in comparatively shallow valleys. These are more or less completely graded valleys. The
rivers have little erosional activity to perform.
 The Himalayan rivers are perennial in nature, i.e., water flows throughout the year in these rivers.
These rivers receive water both from the monsoons and snow-melt. While, the Peninsular rivers
receive water only from rainfall and water flows in these rivers in rainy season only. Therefore, these
rivers are seasonal or non-perennial. Hence, statement 1 is correct.
 Himalayan rivers flow across the young fold mountains and are still in a youthful stage, while
Peninsular rivers have been flowing in one of the oldest plateaus of the world and have reached
maturity. Hence, statement 2 is not correct.
 When Himalayan rivers enter the plains, there is a sudden reduction in the speed of flow of water
which forms meanders and shifts their beds. While, in case of Peninsular rivers, the hard rock surface
and non-alluvial character of the plateau permits little scope for the formation of meanders. As such,
the rivers of the Peninsular Plateau follow more or less straight courses. Hence, statement 3 is
correct.

Q 20.C
o Southern Oscillation is a coherent interannual fluctuation of atmospheric pressure over the tropical
Indo-Pacific region. Hence option (c) is the correct answer.
o It is the atmospheric component of a single large-scale coupled interaction called the El Niño/Southern
Oscillation (ENSO). The phase of the Southern Oscillation at a given point in time may be understood
using the Southern Oscillation Index (SOI), which compares the difference in atmospheric pressure over
Australia and Indonesia with that of the eastern South Pacific.

8 www.visionias.in ©Vision IAS


Q 21.C
o Volcanoes are classified on the basis of nature of eruption and the form developed at the surface. Major
types of volcanoes are as follows:
 Shield Volcanoes
 Barring the basalt flows, the shield volcanoes are the largest of all the volcanoes on the earth. The
Hawaiian volcanoes are the most famous examples. These volcanoes are mostly made up of
basalt, a type of lava that is very fluid when erupted. For this reason, these volcanoes are not
steep. They become explosive if somehow water gets into the vent; otherwise, they are
characterised by low-explosivity. The upcoming lava moves in the form of a fountain and throws
out the cone at the top of the vent and develops into cinder cone. Hence option (c) is the correct
answer.
 Composite Volcanoes
 These volcanoes are characterised by eruptions of cooler and more viscous lavas than basalt.
These volcanoes often result in explosive eruptions. Along with lava, large quantities of
pyroclastic material and ashes find their way to the ground. This material accumulates in the
vicinity of the vent openings leading to formation of layers, and this makes the mounts appear as
composite volcanoes.
 Caldera
 These are the most explosive of the earth‘s volcanoes. They are usually so explosive that when
they erupt they tend to collapse on themselves rather than building any tall structure. The
collapsed depressions are called calderas. Their explosiveness indicates that the magma chamber
supplying the lava is not only huge but is also in close vicinity.
 Flood Basalt Provinces
 These volcanoes outpour highly fluid lava that flows for long distances. Some parts of the world
are covered by thousands of sq. km of thick basalt lava flows. There can be a series of flows with
some flows attaining thickness of more than 50 m. Individual flows may extend for hundreds of
km. The Deccan Traps from India, presently covering most of the Maharashtra plateau, are a
much larger flood basalt province. It is believed that initially the trap formations covered a much
larger area than the present.
 Mid-Ocean Ridge Volcanoes
 These volcanoes occur in the oceanic areas. There is a system of mid-ocean ridges more than
70,000 km long that stretches through all the ocean basins. The central portion of this ridge
experiences frequent eruptions.

Q 22.A
o Types of Earthquakes:
 The most common ones are tectonic earthquakes. These are generated due to the sliding of rocks
along a fault plane.
 A special class of tectonic earthquake is sometimes recognized as a volcanic earthquake. However,
these are confined to areas of active volcanoes.
 In the areas of intense mining activity, sometimes the roofs of underground mines collapse causing
minor tremors. These are called collapse earthquakes.
 Ground shaking may also occur due to the explosion of chemical or nuclear devices. Such tremors are
called explosion earthquakes.
 The earthquakes that occur in the areas of large reservoirs are referred to as reservoir-induced
earthquakes.
o Tsunami is the effect of an earthquake, not a cause of it.

9 www.visionias.in ©Vision IAS


Q 23.B
o Dry hot deserts are good places for sand dune formation. Obstacles to initiate dune formation are equally
important. There can be a great variety of dune forms.

o Crescent-shaped dunes called barchans with the points or wings directed away from wind direction
i.e., downwind, form where the wind direction is constant and moderate and where the original surface
over which sand is moving is almost uniform. Hence, statement 1 is not correct.
o Parabolic dunes form when sandy surfaces are partially covered with vegetation. That means parabolic
dunes are reversed barchans with wind direction being the same.
o Seif is similar to barchan with a small difference. Seif has only one wing or point. This happens
when there is shift in wind conditions. The lone wings of seifs can grow very long and high. Hence,
statement 2 is correct.
o Longitudinal dunes form when the supply of sand is poor and wind direction is constant. They
appear as long ridges of considerable length but low in height. Hence, statement 3 is correct.
o Transverse dunes are aligned perpendicular to the wind direction. These dunes form when the wind
direction is constant and the source of sand is an elongated feature at right angles to the wind direction.
They may be very long and low in height.
o When sand is plenty, quite often, the regular shaped dunes coalesce and lose their individual
characteristics.
o Most of the dunes in the deserts shift and a few of them will get stabilised especially near human
habitations.

10 www.visionias.in ©Vision IAS


Q 24.A

o A catchment is an area of land where water collects when it rains, often bounded by hills. As the water
flows over the landscape it finds its way into streams and down into the soil, eventually feeding the river.
Some of this water stays underground and continues to slowly feed the river in times of low rainfall.

Q 25.A
o The sources of information about the interior of the earth are divided into direct sources and indirect
sources:
 Direct Sources
 The most easily available solid earth material is surface rock or the rocks we get from mining
areas. Gold mines in South Africa are as deep as 3 - 4 km.
 Volcanic eruption forms another source of obtaining direct information. As and when the molten
material (magma) is thrown onto the surface of the earth, during volcanic eruption it becomes
available for laboratory analysis.
 Indirect Sources
 Mining activity provides us with information about temperature and pressure which increases
with the increasing distance from the surface towards the interior in deeper depths.
 Another source of information are the meteors that at times reach the earth. However, it may be
noted that the material that becomes available for analysis from meteors, is not from the interior
of the earth. The material and the structure observed in the meteors are similar to that of the
earth.
 The other indirect sources include gravitation, magnetic field, and seismic activity
(earthquakes). Seismic activity helps us in analysing the composition of the interior of the earth
based on the seismic waves.

11 www.visionias.in ©Vision IAS


Q 26.D
o The major factors of soil formation are the nature of the parent rock and climatic factors. Other factors are
the topography, role of organic material and time taken for the composition of soil formation. All these
differ from place to place.
 Parent Rock: Determines colour, texture, chemical properties mineral, content, permeability
 Climate: Temperature, Rainfall influence rate of weathering and humus formation
 Time: Determines the thickness of soil profile
 Flora, Fauna and Micro-organism: Affect the rate of humus formation
 Relief: Altitude and slope, determine the accumulation of the soil

Q 27.B
o From the map given below, it can be seen that the order of rivers (tributaries of India) from south to north
is-
 Sutlej
 Beas
 Ravi
 Chenab
 Jhelum
o Hence, option (b) is the correct answer.

Q 28.D
o The gravitation force (g) is not the same at different latitudes on the surface. It is greater near the poles
and less at the equator. This is because of the distance from the centre at the equator being greater than
that at the poles.
o The gravity values also differ according to the mass of material. The uneven distribution of mass of
material within the earth influences this value. The reading of the gravity at different places is influenced
by many other factors. These readings differ from the expected values. Such a difference is called
gravity anomaly. Hence, option (d) is the correct answer.

Q 29.A
o The column of atmosphere is divided into five different layers depending upon the temperature condition.
They are: troposphere, stratosphere, mesosphere, thermosphere and exosphere.
12 www.visionias.in ©Vision IAS
o Statement 1 is correct: The troposphere is the lowermost layer of the atmosphere. Its average height is 13
km and extends roughly to a height of 8 km near the poles and about 18 km at the equator. Thickness of
the troposphere is greatest at the equator because heat is transported to great heights by strong
convectional currents. This layer contains dust particles and water vapour. All changes in climate and
weather take place in this layer. The temperature in this layer decreases at the rate of 1°C for every 165m
of height. This is the most important layer for all biological activity.
o Statement 2 is correct: The zone separating the tropsophere from stratosphere is known as the
tropopause. The air temperature at the tropopause is about minus 800 degree C over the equator
and about minus 45 degree C over the poles. The temperature here is nearly constant, and hence, it is
called the tropopause. The stratosphere is found above the tropopause and extends up to a height of 50
km. One important feature of the stratosphere is that it contains the ozone layer. This layer absorbs ultra-
violet radiation and shields life on the earth from intense, harmful form of energy.
o Statement 3 is not correct: The mesosphere lies above the stratosphere, which extends up to a height of
80 km. In this layer, once again, temperature starts decreasing with the increase in altitude and
reaches up to minus 100°C at the height of 80 km. The upper limit of mesosphere is known as the
mesopause.
o The thermosphere is the layer in the Earth's atmosphere directly above the mesosphere and below the
exosphere. It contains electrically charged particles known as ions, and hence, it is a part of region known
as the ionosphere (Ionosphere includes the thermosphere and parts of the mesosphere and exosphere
which are ionized by solar radiation). Radio waves transmitted from the earth are reflected back to the
earth by this layer. Temperature here starts increasing with height.
o The uppermost layer of the atmosphere above the thermosphere is known as the exosphere. This is the
highest layer but very little is known about it. Whatever contents are there, these are extremely rarefied in
this layer, and it gradually merges with the outer space.
o Although all layers of the atmosphere must be exercising influence on us, geographers are concerned with
the first two layers of the atmosphere.

Q 30.C
o Pediplains: Landscape evolution in deserts is primarily concerned with the formation and extension
of pediments. Gently inclined rocky floors close to the mountains at their foot, with or without a
thin cover of debris, are called pediments. Such rocky floors form through the erosion of mountain
front through a combination of lateral erosion by streams and sheet flooding. Erosion starts along the
steep margins of the landmass or the steep sides of the tectonically controlled steep incision feature over
the landmass. Once, pediments are formed with a steep wash slope followed by a cliff or free face above
it, the steep wash slope and free face retreat backwards. This method of erosion is termed as a parallel
retreat of slopes through backwasting. So, through parallel retreat of slopes, the pediments extend
backwards at the expense of mountain front, and gradually, the mountain gets reduced leaving an
inselberg which is a remnant of the mountain. That‘s how the high relief in desert areas is reduced to low
featureless plains called pediplains. Hence, pair 1 is not correctly matched.
o Doline: Small to medium sized round to sub-rounded shallow depressions called swallow holes form
on the surface of limestones through solution by the action of groundwater. Sinkholes are very
common in limestone/karst areas. A sinkhole is an opening more or less circular at the top and funnel-
shapped towards the bottom with sizes varying in area from a few sq. m to a hectare and with depth from
a less than half a metre to thirty metres or more. Some of these form solely through solution action
(solution sinks) and others might start as solution forms first and if the bottom of a sinkhole forms the roof
of a void or cave underground, it might collapse leaving a large hole opening into a cave or a void below
(collapse sinks). Quite often, sinkholes are covered up with soil mantle and appear as shallow water pools.
Anybody stepping over such pools would go down like it happens in quicksands in deserts. The term
doline is sometimes used to refer the collapse sinks. Hence, pair 2 is correctly matched.
o Peneplain: In the early stages of the running water regime, down-cutting dominates during which
irregularities such as waterfalls and cascades will be removed. In the middle stages, streams cut their
beds slower, and lateral erosion of valley sides becomes severe. Gradually, the valley sides are reduced to
lower and lower slopes. The divides between drainage basins are likewise lowered until they are almost
completely flattened leaving finally, a lowland of faint relief with some low resistant remnants called
monadnocks standing out here and there. This type of plain forming as a result of stream erosion is called
a peneplain (an almost plain). Hence, pair 3 is not correctly matched.

13 www.visionias.in ©Vision IAS


Q 31.B
o The current systems of the Indian Ocean are largely controlled and modified by landmasses and
monsoon winds. Indian Ocean is surrounded by the Indian subcontinent, Africa and Australia. This
location does not present most favourable conditions for the development of consistent system of ocean
currents. The currents in the northern Indian Ocean change their flow direction twice a year due to
north-east and south-west monsoon winds.
o In summer from June to October, when the dominant wind is the South-West Monsoon, the currents are
blown from a south-westerly direction as the SouthWest Monsoon Drift. This is reversed in
winter, beginning from December, when the North-East Monsoon blows the currents from the north-east
as the North-East Monsoon Drift. The currents of the North Indian Ocean, demonstrate most
convincingly the dominant effects of winds on the circulation of ocean currents.

Q 32.A
o Break in the Monsoon: During the south-west monsoon period after having rained for a few days, if rain
fails to occur for one or more weeks, it is known as break in the monsoon. Hence statement 1 is not
correct.
o These dry spells are quite common during the rainy season. These breaks occurring in different regions
are due to different reasons:
 In northern India, rains are likely to fail if the rain-bearing storms are not very frequent along the
monsoon trough or the ITCZ over this region. Hence statement 2 is correct.
 Over the west coast, the dry spells are associated with days when winds blow parallel to the coast.
o During break in the monsoon, the trough shifts closer to the foothills of Himalayas or sometimes not
visible at all.
o Middle of August is most prone to ‗breaks‘ and that too longer breaks. Consequently, Northeast and parts
of South India receive good showers while rest of the country remains mainly dry.

Q 33.C
o Physical or mechanical removal of materials by moving groundwater is insignificant in developing
landforms. That is why the results of the work of groundwater cannot be seen in all types of rocks.
o But in rocks like limestones or dolomites rich in calcium carbonate, the surface water as well as
groundwater through the chemical process of solution and precipitation deposition develop varieties of
landforms. These two processes of solution and precipitation are active in limestones or dolomites
occurring either exclusively or interbedded with other rocks.
o Any limestone or dolomitic region showing typical landforms produced by the action of
groundwater through the processes of solution and deposition is called Karst topography after the
typical topography developed in limestone rocks of Karst region in the Balkans adjacent to Adriatic
sea. Hence statements 1 and 2 are correct.

Q 34.D
o Both statements 1 and 2 are not correct: A Volcanic region may be strewn with solid materials that
were hurled from the vent of the Volcano. The very fine particles are the volcanic dust which may be shot
so high into the sky that it travels around the world several times before it eventually comes to rest. The
dust or ash falls as ‗black snow‘ and can bury houses and people. The coarser fragmental rocks are
collectively called pyroclasts and include cinder or lapilli, scoria, pumice and volcanic bombs.

Q 35.A
o Assuming sea level to be constant, two types of coasts are considered to explain the concept of evolution
of coastal landforms: (i) high, rocky coasts (submerged coasts); (ii) low, smooth and gently sloping
sedimentary coasts (emerged coasts).
o Along the high rocky coasts, the rivers appear to have been drowned with a highly irregular coastline. The
coastline appears highly indented with the extension of water into the land where glacial valleys (fjords)
are present. The hillsides drop off sharply into the water. Shores do not show any depositional landforms
initially. Erosion features dominate.
o Along low sedimentary coasts, the rivers appear to extend their length by building coastal plains and
deltas. The coastline appears smooth with occasional incursions of water in the form of lagoons and tidal
creeks. The land slopes gently into the water. Marshes and swamps may abound along the coasts.
Depositional features dominate.
o The west coast of our country is a high rocky retreating coast. Erosional forms dominate in the west
coast.
14 www.visionias.in ©Vision IAS
o The east coast of India is a low sedimentary coast. Depositional forms dominate in the east coast.
o Hence only statement 1 is correct and statement 2 is not correct.

Q 36.B
o Statement 1 is not correct: The earthquake events are scaled either according to the magnitude or
intensity of the shock. The magnitude scale is known as the Richter scale. The magnitude relates to the
energy released during the quake. The intensity scale is named after Mercalli, an Italian
seismologist. It takes into account the visible damage caused by the event. The effect of an earthquake on
the Earth's surface is called the intensity. The intensity scale consists of a series of certain key responses
such as people awakening, movement of furniture, damage to chimneys, and finally - total destruction.
o Statement 2 is correct: In theory, the Richter scale has no upper limit, but, in practice, no earthquake has
ever been registered on the scale above magnitude 8.6 (That was the Richter magnitude for the Chile
earthquake of 1960). The range of intensity (Mercalli) scale is from 1-12.

Q 37.D
o The hypothesis of seafloor spreading was forwarded by Harry Hammond Hess. According to the
hypothesis of seafloor spreading, constant eruptions at the crest of oceanic ridges caused the rupture of the
oceanic crust and the new lava wedges into it, pushing the oceanic crust on either side. Thus the rocks
equidistant on either side of the crest of mid-oceanic ridges show remarkable similarities in terms of
the period of formation, chemical compositions and magnetic properties. Hence statement 1 is
correct.
o The ocean floor thus spreads. Further, the ocean floor that gets pushed due to volcanic eruptions at the
crest, sinks down at the oceanic trenches and gets consumed. Hence the oceanic crust is simultaneously
formed at the mid-oceanic ridge and consumed at oceanic trenches. Thus, ocean crust rocks are much
younger than the continental rocks. In addition, the deep trenches have deep-seated earthquake
occurrences while in the mid-oceanic ridge areas, the quake foci have shallow depths. Hence
statements 2 and 3 are correct.

15 www.visionias.in ©Vision IAS


Q 38.C
o India is flanked by the Indian Ocean on three sides in the south and girdled by a high and continuous
mountain-wall in the north.
o As compared to the landmass, water heats up or cools down slowly. This differential heating of land
and sea creates different air pressure zones in different seasons in and around the Indian subcontinent.
o The difference in air pressure causes a reversal in the direction of monsoon winds.
o Hence both the statements are correct.

Q 39.C
o The Agulhas Current is the western boundary current of the South-West Indian Ocean. It flows down
the east coast of Africa. The source water at its northern end is derived from Mozambique channel eddies
and the East Madagascar Current, but the greatest source of water is recirculation in the southwest Indian
Ocean sub-gyre. Hence pair 1 is correctly matched.
o The Irminger Current is a north Atlantic ocean current setting westward off the southwest coast of
Iceland. It is composed of relatively warm and saline waters from the eastern North Atlantic that are fed
by the North Atlantic Drift. The Irminger Current is part of the North Atlantic subpolar gyre. Hence pair
2 is not correctly matched.
o The Humboldt Current, also called the Peru Current, is a cold, low-salinity ocean current that flows
north along the western coast of South America. Hence pair 3 is correctly matched.

Q 40.D

16 www.visionias.in ©Vision IAS


o Karakoram Range : It is a large mountain range spanning the borders between Pakistan, India and
China, located in the regions of Gilgit-Baltistan (Pakistan), Ladakh (India), and Xinjiang (China). It
is also known as Krishnagiri which is situated in the northern most range of the Trans-Himalayan
ranges.
 It is the northwestern extension of the Himalaya.
 This range is home to many peaks over five miles in height, including K2 (Mount Godwin Austen),
the second highest peak in the world (8,611 m/28,251 ft), and three other of the Earth's fourteen
8,000-meter peaks — Gasherbrum I at 8,068 m. (26,471 ft), Broad Peak at 8,047 m. (26,402 ft),
and Gasherbrum II at 8,035 m. (26,363 ft).
 It is heavily glaciated. About 28-50% of these mountains are glaciated, much more than the
Himalayan average of only 8-12%.The Siachen Glacier at 44 miles long and the Biafo Glacier at 39
miles long are the world's second and third longest glaciers outside the Arctic and Antarctic areas.
o Ladakh Range: It lies to the north of the Leh and is an important part of the Trans-Himalayan range
that merges with the Kailash range in Tibet. The important passes that lie to the north east of Leh are-
Kardung La, and Digar La.
o Zaskar/Zanskar Range: It covers an area of some 7,000 square kilometres (2,700 square miles), at a
height of between 3,500 and 7,000 metres. High mountain ranges lie on both sides of the Doda and
Kurgiakh valleys, which run north-west to south-east. To the south-west is the Great Himalayan Range
which separates Zanskar from the Kisthwar and Chamba basins. To the north-east lies the Zanskar
Range, which separates Zanskar from Ladakh. The only outlet for the whole Zanskar hydrographic
system is thus the Zanskar river, which cuts a deep and narrow gorge through the Zanskar range.
o Pir Panjal Range: It is a group of mountains in the inner Himalayan region, running from east-
southeast (ESE) to west-northwest (WNW) across the Indian states of Himachal Pradesh and
Jammu and Kashmir and Pakistan administered Kashmir, where the average elevation varies from
1,400 m (4,600 ft) to 4,100 m (13,500 ft).
 It is the largest range of the lower Himalayas. Near the bank of the Sutlej river, it dissociates itself
from the Himalayas and forms a divide between the rivers Beas and Ravi on one side and the
Chenab on the other. The famous Murree and Galliat mountains are also located in this range.
 Deo Tibba and Indrasan are two important peaks at the eastern end of the mountain range.
 The hill station of Gulmarg in Kashmir lies in this range.
 Passes:
 The Pir Panjal pass lies to the west of Srinagar.
 The Banihal pass lies at the head of the Vitasta river at the southern end of the Kashmir valley.
Banihal and Qazigund lie on either side of the pass.
 The Sinthan pass connects Jammu and Kashmir with Kishtwar.
 Rohtang La is a mountain pass on the eastern Pir Panjal range connecting Manali in the Kullu
Valley to Keylong in the Lahaul Valley.

Q 41.A
o The moon is the only natural satellite of the earth. There have been several attempts to describe the origin
of the moon.
o In 1838, Sir George Darwin suggested that initially, the earth and the moon formed a single rapidly
rotating body. The whole mass became a dumb-bell-shaped body and eventually, it broke. It was also
suggested that the material forming the moon was separated from what we have at present the depression
occupied by the Pacific Ocean.
o However, these explanations came under severe scrutiny. This led to the proposal of ―the big splat‖
theory in the 1970s. Hence, option (a) is the correct answer.
o According to this theory, an object with the mass of Mars delivered a glancing blow to Earth, launching
large amounts of rock into an orbiting ring that coalesced to form the Moon. Most of the Moon would
have been made of material from the impactor‘s mantle. The angle of the impact gave the Earth-Moon
system its current angular momentum.
o However, over the years problems with even this theory have emerged. For e.g., astronomers haven‘t
found any trace of the impactor‘s chemical makeup. Instead, measurements show that the Moon and Earth
are made of exactly the same stuff.
Q 42.C
o The Earth is made up of several concentric layers. The outer layer is called Crust which comprises of two
distinct parts - Continental and Oceanic Crust. The Continental Crust consists of Granitic rocks and its
main mineral contents are Silicon and Alumina. Thus, it is collectively referred to as Sial. Hence, pair 1
is not correctly matched.
17 www.visionias.in ©Vision IAS
o The Oceanic Crust is a continuous Zone of denser basaltic rocks forming the Ocean floors, comprising
mainly Silica, iron and magnesium. Thus, it is referred to as Sima. Hence, pair 2 is not correctly
matched.
o The layer below the Crust is called Mantle. It is composed of very dense rocks rich in Olivine. The
interior layer is the Core and is mainly made up of iron and some nickel, and is called nife. Hence, pair
3 and 4 are correctly matched.

Q 43.D
o The lava that is released during volcanic eruptions on cooling develops into igneous rocks. The cooling
may take place either on reaching the surface or also while the lava is still in the crustal portion. The lava
that cools within the crustal portions assumes different forms. These forms are called intrusive
forms. They are:
 Batholiths
 A large body of magmatic material that cools in the deeper depth of the crust develops in the form
of large domes. They appear on the surface only after the denudational processes remove the
overlying materials. Hence, option 1 is correct.
 Lacoliths
 These are large dome-shaped intrusive bodies with a level base and connected by a pipe-like
conduit from below. It resembles the surface volcanic domes of composite volcano, only these are
located at deeper depths. It can be regarded as the localised source of lava that finds its way to the
surface.
 Lapolith, Phacolith and Sills
 As and when the lava moves upwards, a portion of the same may tend to move in a horizontal
direction wherever it finds a weak plane. It may get rested in different forms. In case it develops
into a saucer shape, concave to the sky body, it is called lapolith. A wavy mass of intrusive rocks,
at times, is found at the base of synclines or at the top of anticline in folded igneous country. Such
wavy materials have a definite conduit to source beneath in the form of magma chambers
(subsequently developed as batholiths). These are called the phacoliths. The near horizontal
bodies of the intrusive igneous rocks are called sill or sheet, depending on the thickness of the
material. The thinner ones are called sheets while the thick horizontal deposits are called
sills. Hence, options 2 and 3 are correct.
 Dykes
 When the lava makes its way through cracks and the fissures developed in the land, it solidifies
almost perpendicular to the ground. It gets cooled in the same position to develop a wall-like
structure. Such structures are called dykes.
o The formation of landforms by cooling of lava on reaching the surface are called Extrusive landforms.
Some of them include Lava Mesa, Lava Plateau, Cinder Cone etc. Hence, option 4 is correct.

Q 44.A
o India shares its land boundaries with Pakistan and Afghanistan in the northwest, China (Tibet), Nepal and
Bhutan in the north and Myanmar and Bangladesh in the east. Our southern neighbours across the sea
consist of the two island countries, namely Sri Lanka and Maldives. Sri Lanka is separated from India by
a narrow channel of sea formed by the Palk Strait and the Gulf of Mannar, while Maldives Islands are
situated to the south of the Lakshadweep Islands.
18 www.visionias.in ©Vision IAS
o Name of the country: Length of the border (in km):
 Bangladesh: 4,096.7
 China: 3,488
 Pakistan: 3,323
 Nepal: 1,751
 Myanmar: 1,643
 Bhutan: 699
 Afghanistan: 106

Q 45.C
o Both the statements 1 and 2 are correct : The western coastal plains are narrow in the middle and get
broader towards north and south. The rivers flowing through this coastal plain do not form any delta.
o On the East Coast, There are well-developed deltas here, formed by the rivers flowing eastward in to the
Bay of Bengal. These include the deltas of the Mahanadi, the Godavari, the Krishna and the Kaveri.
o Deltas are wetlands that form as rivers empty their water and sediment into another body of water, such as
an ocean, lake, or another river. Although very uncommon, deltas can also empty into land. A river moves
more slowly as it nears its mouth, or end. This causes sediment, solid material carried downstream
by currents, to fall to the river bottom.
o The hard nature of rocks of Western ghat doesn't facilitate Western flowing rivers to widen their mouth
into sea, so these rivers form Estuaries.On the Other hand, Eastern rivers usually flow by breaking
through hills and mountain ranges and flow with less velocity before entering into sea. Subsequently, the
accumalated silt and sediments which they have been carrying along is deposited before entering into the
sea, thus Deltas are more prominent on the east coast.

Q 46.C
o An ocean gyre is a large system of circular ocean currents formed by global wind patterns and forces
created by Earth‘s rotation. The movement of the world‘s major ocean gyres helps drive the ocean
conveyor belt. The ocean conveyor belt circulates ocean water around the entire planet.
o The Kuroshio Current is the warm western boundary current of the North Pacific's subtropical
gyre. Hence statement 3 is correct.
o The Kuroshio Current begins off the east coast of Philippines and Taiwan and flows northeastward past
Japan, where it merges with the easterly drift of the North Pacific Current. Hence statement 1 is not
correct.
o It is analogous to the Gulf Stream in the Atlantic Ocean. Its warm waters are carried polewards as the
North Pacific Drift keeping the ports of Alaskan coast ice-free in winter. Hence statement 2 is correct.

Q 47.D
o Chota Nagpur plateau represents the north eastern projection of the Indian Peninsula. It covers an area
of over 87 thousand sq km. It covers much of Jharkhand state as well as adjacent parts of Odisha,
West Bengal, Bihar, and Chhattisgarh. Son river flows in the north-west of the plateau and joins the
Ganga. The average elevation of the plateau is 700 meter above sea level. Hence Statement 1 is not
correct.
o The Eastern Ghats or Pūrva Ghaṭ , also known as Mahendra Parvatam in the south, is a discontinuous
range of mountains along India's eastern coast. Starting at West Bengal, Eastern Ghats pass through states
like Orissa, Andhra Pradesh, Telengana and Tamil Nadu to the south passing through some parts
of Karnataka. They are eroded and cut through by the four major rivers of peninsular India, known as the
Godavari, Mahanadi, Krishna, and Kaveri. Hence Statement 2 is not correct.

Q 48.D
o The earth after being heated by insolation transmits the heat to the atmospheric layers near to the earth in
a long waveform. The air in contact with the land gets heated slowly and the upper layers in contact with
the lower layers also get heated. This process is called conduction.
o Statement 1 is correct: Conduction takes place when two bodies of unequal temperature are in contact
with one another; there is a flow of energy from the warmer to a cooler body. The transfer of heat
continues until both the bodies attain the same temperature or the contact is broken. Conduction is
important in heating the lower layers of the atmosphere.
o Statement 2 is correct: The air in contact with the earth rises vertically on heating in the form of currents
and further transmits the heat of the atmosphere. This process of vertical heating of the atmosphere is
known as convection. The convective transfer of energy is confined only to the troposphere.
19 www.visionias.in ©Vision IAS
o Statement 3 is correct: The transfer of heat through the horizontal movement of air is called advection.
The horizontal movement of the air is relatively more important than the vertical movement. In middle
latitudes, most of diurnal (day and night) variations in daily weather are caused by advection alone.
In tropical regions particularly in northern India during the summer season, local winds called ‗loo‘ is the
outcome of advection process.

Q 49.D
o Surface Pressure and Winds during the summer season in the Indian Subcontinent
 As the summer sets in and the sun shift northwards, the wind circulation over the subcontinent
undergoes a complete reversal at both, the lower as well as the upper levels.
 By the middle of July, the low-pressure belt nearer the surface [termed as Inter-Tropical
Convergence Zone (ITCZ)] shifts northwards, roughly parallel to the Himalayas between 20° N
and 25° N.
 By this time, the westerly jet stream withdraws from the Indian region.
 The interrelationship between the northward shift of the equatorial trough (ITCZ) and the
withdrawal of the westerly jet stream from over the North Indian Plain.
 It is generally believed that there is a cause and effect relationship between the two.
 The ITCZ being a zone of low pressure attracts inflow of winds from different directions. The
maritime tropical airmass (mT) from the southern hemisphere, after crossing the equator, rushes to the
low pressure area in the general southwesterly direction. It is this moist air current which is popularly
known as the southwest monsoon.
o Hence, all the statements are correct.

Q 50.B
o It is always advisable to adopt area-specific measures to deal with landslides. Restriction on the
construction and other developmental activities such as roads and dams, limiting agriculture to valleys and
areas with moderate slopes, and control on the development of large settlements in the high vulnerability
zones, should be enforced.
o This should be supplemented by some positive actions like promoting large-scale afforestation
programmes and construction of bunds to reduce the flow of water. Terrace farming should be
encouraged in the north-eastern hill states where Jhumming(Slash and Burn/Shifting Cultivation) is still
prevalent.
o Jhumming is often considered responsible for causing soil erosion, triggering landslide, flash floods
and thereby degrading the primary land resource.

Q 51.D
o A large number of hypotheses were put forth by different philosophers and scientists regarding the origin
of the earth. One of the earlier and popular arguments was by German philosopher Immanuel Kant.
Mathematician Laplace revised it in 1796. It is known as the Nebular Hypothesis. Hence statement 3 is
correct.
o The hypothesis considered that the planets were formed out of a cloud of material associated with a
youthful sun, which was slowly rotating. Hence statement 1 is correct. Later in 1900, Chamberlain and
Moulton considered that a wandering star approached the sun. As a result, a cigar-shaped extension of the
material was separated from the solar surface. As the passing star moved away, the material separated
from the solar surface continued to revolve around the sun and it slowly condensed into planets.
o Nebular Hypothesis is used for explaining the origin of the Solar system. Hence statement 2 is correct.

Q 52.D
o Statement 1 is not correct: The upper portion of the mantle is called asthenosphere. The word 'astheno'
means weak. It is a layer of solid rock that has so much pressure and heat the rocks can flow like a liquid.
It is considered to be extending up to 400 km.
o Statement 2 is not correct: It is the main source of magma that finds its way to the surface during
volcanic eruptions. It has a density higher than the crust‘s (3.4 g/cm 3 ). The crust and the uppermost parts
of the mantle are called lithosphere. Aesthenosphere does not form a part of the lithosphere.

20 www.visionias.in ©Vision IAS


*Note- The mesophere in geology is not to be confused with the atmospheric mesosphere (which lies
directly above the stratosphere). In geology, mesosphere is a part of the mantle which lies below the
asthenosphere. The upper boundary of mesosphere is defined by a sharp increase in the velocities of
the seismic waves.
Q 53.A
o Statement 1 is correct: In dry climates, because of high temperature, evaporation exceeds precipitation
and hence groundwater is brought up to the surface by capillary action and in the process, the water
evaporates leaving behind salts in the soil. Such salts form into a crust on the soil known as hardpans.
o Statement 2 is correct: Salinization leads to intense water logging which means a decrease in the
porosity of soil and hence the water-holding capacity. And in the long term, makes the soil impermeable.
o Statement 3 is not correct: An estuary is a body of water, partially enclosed by land, where saltwater
from the ocean and freshwater from the land can mix. Estuaries can be found in coastal areas of the ocean
worldwide.
o The salinity of water in the surface layer of oceans depend mainly on evaporation and precipitation.
o Surface salinity is greatly influenced in coastal regions by the freshwater flow from rivers, and in polar
regions by the processes of freezing and thawing of ice.
o Wind also influences the salinity of an area by transferring water to other areas.
o The ocean currents contribute to the salinity variations. Salinity, temperature and density of water are
interrelated. Hence, any change in the temperature or density influences the salinity of an area.
Q 54.C
o Equatorial type of climate is associated with all the given features given below:
 The most outstanding feature of the equatorial climate is its great uniformity of temperature
throughout the year.
 There is no winter.
 Cloudiness and heavy precipitation help to moderate the daily temperature, so that even at the
equator itself, the climate is not unbearable. In addition, regular land and sea breezes assist in
maintaining a truly equable climate.
 The diurnal range of temperature is small, and so is the annual range. High temperature and
abundant rainfall in the equatorial regions support a luxuriant type of vegetation i.e. the Tropical Rain
Forests.
 The equatorial vegetation comprises a multitude of evergreen trees that yield tropical hardwood,
e.g. mahogany, ebony, etc. It supports a luxuriant type of vegetation – the tropical rain forest.
Amazon tropical rain forest is known as Selvas. Trees of single species are very scarce in such
vegetation. There are smaller palm trees, climbing plants like the lianas or rattan which may be
hundreds of feet long and epiphytic and parasitic plants that live on other plants. Under the trees grow
a wide variety of ferns, orchids and lalang. The tallest trees attain a height of over 150 feet.
Q 55.B
o Statements 1 and 2 are not correct: Mass movements transfer the mass of rock debris down the
slopes under the direct influence of gravity. That means, air, water or ice do not carry debris with them
from place to place but on the other hand the debris may carry with it air, water or ice. Gravity exerts its
force on all matter, both bedrock and the products of weathering. So, weathering is not a pre-requisite
for mass movement though it aids mass movements. Mass movements are very active over
weathered slopes rather than over unweathered materials.
o Mass movements are aided by gravity and no geomorphic agent like running water, glaciers, wind, waves
and currents participate in the process of mass movements. That means mass movements do not come
21 www.visionias.in ©Vision IAS
under erosion though there is a shift (aided by gravity) of materials from one place to another. Materials
over the slopes have their own resistance to disturbing forces and will yield only when force is greater
than the shearing resistance of the materials.
o Statement 3 is correct: Weak unconsolidated materials, thinly bedded rocks, faults, steeply dipping beds,
vertical cliffs or steep slopes, abundant precipitation and torrential rains and scarcity of
vegetation etc., favour mass movements.

Q 56.D
o Statement 1 is not correct: The solar output received at the top of the atmosphere varies slightly in a
year due to the variations in the distance between the earth and the sun. During its revolution around the
sun, the earth is farthest from the sun (152 million km) on 4th July. This position of the earth is called
aphelion. On 3rd January, the earth is the nearest to the sun (147 million km). This position is called the
perihelion. Therefore, the annual insolation received by the earth on 3rd January is slightly more
than the amount received on 4th July. However, the effect of this variation in the solar output is masked
by other factors like the distribution of land and sea and the atmospheric circulation.
o Statement 2 is correct: The insolation received at the surface varies from about 320 Watt/m2 in the
tropics to about 70 Watt/m2 in the poles. Maximum insolation is received over the subtropical deserts,
where the cloudiness is the least. The Equator receives comparatively less insolation than the tropics .
o Statement 3 is not correct: Generally, at the same latitude, the insolation is more over the continent
than over the oceans. In winter, the middle and higher latitudes receive less radiation than in summer.

Q 57.D
o From the Sub-Tropical High-Pressure Belts, winds blow towards the Temperate Low-Pressure Belts as the
variable Westerlies. Under the effect of the Coriolis Force, they become the South-Westerlies in the
northern hemisphere and the North-Westerlies in the southern hemisphere.
o They are more variable in the northern hemisphere. but they play a valuable role in carrying warm
equatorial waters and winds to western coasts of temperate lands. This warming effect and other local
pressure differences have resulted in a very variable climate in the temperate zones, dominated by the
movements of cyclones and anticyclones.
o In the southern hemisphere where there is a large expanse of ocean, 40 degrees S to 60 degree S,
Westerlies blow with much greater force and regularity throughout the year. They bring much
precipitation to the western coasts of continents. The weather is damp and stormy. It is thus usual for
seafarers to refer to the Westerlies as the Roaring Forties, Furious Fifties and Shrieking Sixties,
according to the varying degree of storminess in the latitudes which they blow.

Q 58.D
o The following countries share their borders with the Mediterranean Sea: Spain, France, Monaco, Italy,
Malta, Slovenia, Croatia, Bosnia and Herzegovina, Montenegro, Albania, Greece, Turkey, Cyprus,
Syria, Lebanon, Israel, Egypt, Libya, Tunisia, Algeria and Morocco.

22 www.visionias.in ©Vision IAS


Q 59.A
o The whole of India has a monsoon type of climate. But the combination of elements of the weather,
however, reveal many regional variations. These variations represent the subtypes of the monsoon
climate. It is on this basis that the climatic regions can be identified. A climatic region has a homogeneous
climatic condition which is the result of a combination of factors. Temperature and rainfall are two
important elements which are considered to be decisive in all the schemes of climatic classification.There
are different schemes of classification of climate. Major climatic types of India based on Koeppen‘s
scheme have been shown below:

Q 60.B
o Deciduous Forests are the most widespread forests in India. They are also called the monsoon forests.
They spread over regions which receive rainfall between 70-200 cm. On the basis of the availability of
water, these forests are further divided into moist and dry deciduous.
o The moist deciduous forests are more pronounced in the regions which record rainfall between 100-200
cm. These forests are found in the northeastern states along the foothills of Himalayas, eastern slopes of
the Western Ghats and Orissa. Teak, sal, shisham, hurra, mahua, amla, semul, kusum, and
sandalwood etc. are the main species of these forests.
o Dry deciduous forest covers vast areas of the country, where rainfall ranges between 70 -100 cm. On the
wetter margins, it has a transition to the moist deciduous, while on the drier margins to thorn forests.
These forests are found in rainier areas of the Peninsula and the plains of Uttar Pradesh and Bihar.As the
dry season begins, the trees shed their leaves completely and the forest appears like a vast grassland with
naked trees all around. Tendu, palas, amaltas, bel, khair, axlewood, etc. are the common trees of these
forests.

Q 61.B

o The ocean floors can be divided into four major divisions: (i) the Continental Shelf; (ii) the Continental
Slope; (iii) the Deep Sea Plain; (iv) the Oceanic Deeps. Besides, these divisions there are also major and
minor relief features in the ocean floors like ridges, hills, sea mounts, guyots, trenches, canyons, etc.
o Continental Shelf: The continental shelf is the extended margin of each continent occupied by relatively
shallow seas and gulfs. It is the shallowest part of the ocean showing an average gradient of 1° or even
less.
23 www.visionias.in ©Vision IAS
o Continental Slope: The continental slope connects the continental shelf and the ocean basins. It begins
where the bottom of the continental shelf sharply drops off into a steep slope.
o Continental Rise: The continental rise is found between the continental slope and the abyssal plain. It
represents the final stage in the boundary between continents and the deepest part of the ocean. At the
bottom of the continental slope, one will find the continental rise, an underwater hill composed of tons of
accumulated sediments.
o Abyssal Plain: The abyssal plain is an underwater plain on the deep ocean floor, usually found at depths
between 3,000 metres and 6,000 metres. It lies between the foot of a continental rise and a mid-ocean
ridge.
o Hence the correct sequence is 4-1-2-3.

Q 62.D
o Monsoon is that axis around which revolves the entire agricultural cycle of India. It plays a pivotal role in
the agrarian economy of India because over three-fourths of the total rain in the country is received during
the southwest monsoon season. Its spatial distribution is also uneven which ranges from 12 cm to more
than 250 cm.
o Variability of rainfall brings droughts or floods every year in some parts of the country. Hence option 2 is
correct.
o Sudden monsoon burst creates problem of soil erosion over large areas in India. The summer rainfall
comes in a heavy downpour leading to considerable run off and soil erosion. Hence option 1 is correct.
o It is because about 64 per cent people of India depend on agriculture for their livelihood and agriculture
itself is based on southwest monsoon. Regional variations in monsoon climate help in growing various
types of crops.
o Agricultural prosperity of India depends very much on timely and adequately distributed rainfall. If it
fails, agriculture is adversely affected particularly in those regions where means of irrigation are not
developed.
o The beginning of the rains sometimes is considerably delayed over the whole or a part of the country. The
rains sometimes end considerably earlier than usual, causing great damage to standing crops and making
the sowing of winter crops difficult. Hence option 3 is correct.

Q 63.D
o Guyot: It is a flat-topped seamount. They show evidence of gradual subsidence through stages to become
flat-topped submerged mountains. Hence, pair 1 is correctly matched.
o Trenches: These areas are the deepest parts of the oceans. The trenches are relatively steep-sided, narrow
basins. They are some 3-5 km deeper than the surrounding ocean floor. They occur at the bases of
continental slopes and along island arcs and are associated with active volcanoes and strong
earthquakes. Hence, pair 3 is correctly matched.
o Mid Oceanic Ridges: A mid-ocean ridge (MOR) is a seafloor mountain system formed by plate tectonics.
It typically has a depth of around 2,600 meters and rises about two kilometres above the deepest portion
of an ocean basin. This feature is where seafloor spreading takes place along a divergent plate boundary.
The rate of seafloor spreading determines the morphology of the crest of the mid-ocean ridge and its
width in an ocean basin. Hence, pair 2 is correctly matched.

Q 64.D
o Statement 1 is correct: Eastern Ghats comprising the discontinuous and low hills are highly eroded by
the rivers such as the Mahanadi, the Godavari, the Krishna, the Kaveri, etc. Some of the important ranges
include the Javadi hills, the Palconda range, the Nallamala hills, the Mahendragiri hills, etc. The Eastern
and the Western Ghats meet each other at the Nilgiri hills.
o Statement 2 is correct: Western Ghats are comparatively higher in elevation and more continuous than
the Eastern Ghats. Their average elevation is about 1,500 m with the height increasing from north to
south. ‗Anaimudi‘ (2,695 m), the highest peak of Peninsular plateau is located on the Anaimalai hills of
the Western Ghats followed by Dodabetta (2,637 m) on the Nilgiri hills. Most of the Peninsular rivers
have their origin in the Western Ghats.
o Statement 3 is correct: Deccan Plateau is bordered by the Western Ghats in the west, Eastern Ghats in
the east and the Satpura, Maikal range and Mahadeo hills in the north. Western Ghats are locally known
by different names such as Sahyadri in Maharashtra, Nilgiri hills in Karnataka and Tamil Nadu and
Anaimalai hills and Cardamom hills in Kerala.

24 www.visionias.in ©Vision IAS


Q 65.C
o Statement 1 is correct: The Jhelum river rises from a deep spring at Vernag, in western Jammu and
Kashmir state. Srinagar, capital city of the state of Jammu and Kashmir is located on the banks of Jhelum
river.
o Statement 2 is correct: Jhelum in the valley of Kashmir is still in its youth stage and yet forms
meanders– a typical feature associated with the mature stage in the evolution of fluvial landform. The
river meanders northwestward from the northern slope of the Pir Panjal Range through the Vale of
Kashmir to Wular Lake at Srinagar, which controls its flow. It meanders due to local base level of
sediments provided by a primeval lake which existed millions of years ago in Kashmir Valley.

Q 66.A
o Jet Streams and Upper Air Circulation during the summer season: The Easterly Jetstream is a unique
and dominant feature of the northern hemispheric summer over southern Asia and northern Africa. The
Easterly Jetstream is found near between 5° and 20°N. It flows from east to west over peninsular India at
6 – 9 km and over the Northern African region. It results in the reversal of upper air circulation patterns
[High pressure switches to low pressure] and leads to the quick onset of monsoons.
o Statement 1 is correct: The easterly jet stream steers the tropical depressions into India. These
depressions play a significant role in the distribution of monsoon rainfall over the Indian subcontinent.
The tracks of these depressions are the areas of the highest rainfall in India.
o Statement 2 is not correct: An easterly jet stream flows over the southern part of the Peninsula in June
and has a maximum speed of 90 km per hour. In August, it is confined to 15°N latitude, and in September
up to 22° N latitudes. The easterlies normally do not extend to the north of 30° N latitude in the upper
atmosphere. Hence, the influence of the easterly jet stream keeps increasing towards the north
during the southwest monsoon period.

Q 67.B
o Tropic of Cancer is an imaginary line, at an angle of 23.50 degrees North from the Equator, that passes
through roughly the middle of India. If you run this imaginary line around Earth, it passes through 17
countries, India being one of them.
o The Tropic of Cancer passes through eight states in India: Gujarat (Jasdan), Rajasthan (Kalinjarh),
Madhya Pradesh (Shajapur), Chhattisgarh (Sonhat), Jharkhand (Lohardaga), West Bengal (Krishnanagar),
Tripura (Udaipur) and Mizoram (Champhai) in that order.
o Mahi River is the only river in India that cuts the Tropic of Cancer twice, first in Madhya Pradesh from
where it flows towards Rajasthan and enters Gujarat where it cuts for the second time. Udaipur in
Tripura is the city nearest to the Tropic of Cancer.

Q 68.C
o Some Famous Local Storms of Hot Weather Season in India are:
 Mango Shower: Towards the end of summer, there are pre-monsoon showers which are a common
phenomena in Kerala and coastal areas of Karnataka. Locally, they are known as mango showers
since they help in the early ripening of mangoes.
 Blossom Shower: With this shower, coffee flowers blossom in Kerala and nearby areas.
 Nor Westers: These are dreaded evening thunderstorms in Bengal and Assam. Their notorious nature
can be understood from the local nomenclature of ‗Kalbaisakhi‘, a calamity of the month of Baisakh.
These showers are useful for tea, jute and rice cultivation. In Assam, these storms are known as
―Bardoli Chheerha‖.
 Loo: Hot, dry and oppressing winds blowing in the Northern plains from Punjab to Bihar with higher
intensity between Delhi and Patna.
 Mustard is a Rabi crop, therefore storms of Hot Weather Season do not assist in its growth.
 Hence option (c) is the correct answer.

Q 69.C
o Statement 1 is correct: The continents that the Equator passes through include Australia and Oceania,
South America, Africa and Asia. This imaginary line that circles the world traverses 13 countries:
 Kiribati in Australia and Oceania;
 Ecuador, Colombia and Brazil in South America;
 Sao Tome and Principe, Gabon, Democratic Republic of Congo, Congo, Kenya, Uganda and Somalia
in Africa and
 Maldives and Indonesia in Asia.
25 www.visionias.in ©Vision IAS
o Statement 2 is correct: The Limpopo river rises in South Africa and flows generally eastwards through
Mozambique to the Indian Ocean. Its length is 1750km. It passes through Tropic of Capricorn twice.
o Other Related Information: The Mahi river in India passes through the Tropic of Cancer twice. The
Congo river in Africa passes through the equator twice.

Q 70.C
o The Indian Oceans winter monsoon (Northeast Monsoon), which lasts from October to April, is less well-
known than its rainy summer equivalent. The dry winter monsoon blows from the northeast. These winds
start in the air above Mongolia and northwestern China.
o Winter monsoons do not cause rainfall as they move from land to the sea. It is because:
 They have little humidity; and
 due to anticyclonic circulation on land, the possibility of rainfall from them reduces.
o So, most parts of India do not have rainfall in the winter season.
o On the other hand, the low temperature generally assists in condensation of water vapors in the
atmosphere. But in case of low humidity, water vapors are inadequate to get precipitated in the form of
rainfall. Winter monsoon causes heavy rainfall in Tamil Nadu. This is because the winds pass over Bay of
Bengal, picking up moisture in the process.

Q 71.C
o Statement 2 is correct: Due to the Coriolis force. winds blowing out from the Sub-Tropical High-
Pressure Belt in the northern hemisphere towards the Equatorial Low become North-East Trade Winds
and those in the southern hemisphere become the South-East Trade Winds. These trade winds are the
most regular of all the planetary winds. They blow with great force and in a constant direction. They were
thus helpful to early traders who depended on the wind when sailing the high seas; hence the name 'trade
winds'. Since they blow from the cooler sub-tropical latitudes to the warmer tropics, they have a great
capacity for holding moisture. In their passage across the open oceans, they gather more moisture
and bring heavy rainfall to the east coasts of continents within tropics.
o As they are off-shore on the west coast. these regions suffer from great aridity and form the Trade Wind
Hot Deserts of the world. e.g. Sahara, Kalahari, Atacama and the Great Australian Deserts.
o Statement 1 is correct: From the Sub-Tropical High-Pressure Belts, winds blow towards the Sub-
Polar/Temperate Low-Pressure Belts as the variable Westerlies. Under the effect of the Coriolis Force,
they become the South-Westerlies in the northern hemisphere and the North-Westerlies in the southern
hemisphere. They are more variable in the northern hemisphere. but they play a valuable role in carrying
warm equatorial waters and winds to western coasts of temperate lands. This warming effect and
other local pressure differences have resulted in a very variable climate in the temperate zones, dominated
by the movements of cyclones and anticyclones.

Q 72.C
o The Brahmaputra, one of the largest rivers of the world, has its origin in the Chemayungdung glacier of
the Kailash range near the Mansarovar lake. From here, it traverses eastward longitudinally for a distance
of nearly 1,200 km in a dry and flat region of southern Tibet, where it is known as the Tsangpo, which
means ‗the purifier.‘ The Rango Tsangpo is the major right bank tributary of this river in Tibet. It emerges
as a turbulent and dynamic river after carving out a deep gorge in the Central Himalayas near Namcha
Barwa (7,755 m). Hence, statement 1 is correct.
o The river emerges from the foothills under the name of Siang or Dihang. It enters India from Arunachal
Pradesh. Flowing southwest, it receives its main left bank tributaries, viz., Dibang or Sikang and Lohit;
thereafter, it is known as the Brahmaputra. Hence, statement 2 is not correct and statement 3 is
correct.
26 www.visionias.in ©Vision IAS
o The Brahmaputra receives numerous tributaries in its 750 km long journey through the Assam valley. Its
major left bank tributaries are the Burhi Dihing and Dhansari (South) whereas the important right bank
tributaries are the Subansiri, Kameng, Manas and Sankosh. The Subansiri which has its origin in Tibet, is
an antecedent river. The Brahmaputra enters into Bangladesh near Dhubri and flows southward. In
Bangladesh, the Tista joins it on its right bank from where the river is known as the Jamuna. It finally
merges with the river Padma, which falls in the Bay of Bengal.

Q 73.C
o Statement 1 is correct: Anticyclones are the opposite of cyclones, with high pressure in the centre and
the isobars far apart. The pressure gradient is gentle and winds are light.
o Statement 2 is correct: Anti-cyclones normally herald fine weather. Skies are clear. The air is
calm and temperatures are high in summer but cold in winter. In winter intense cooling of the lower
atmosphere may result in thick fogs. Anti-cyclonic conditions may last for days or weeks and then fade
out quietly.
o Statement 3 is not correct: Winds in anticyclone' blow outwards and are also subject to deflection, but
they blow clockwise in the northern hemisphere and anticlockwise in the southern hemisphere.

Q 74.A
o Strait of Hormuz lies between Iran, Oman and UAE and connects Persian Gulf with Gulf of Oman and
Arabian sea. It provides only sea passage from persian gulf to open ocean (Arabian Sea). Hence pair 1 is
correctly matched.

o Bab-el Mandeb is between Yemen on the Arabian Peninsula and Djibouti and Eritrea on the Horn of
Africa. It connects Red sea to Gulf of Aden and Indian Ocean. Hence pair 2 is not correctly matched.

o Bosphorous Strait is Between Asian Turkey and European Turkey. It connects Black sea to Sea of tched

27 www.visionias.in ©Vision IAS


o Magellan Strait is in southern Chile separating mainland South America to the north and Tierra del
Fuego to the south and is a natural passage between Atlantic and Pacific oceans. Hence pair 4 is not
correctly matched.

Q 75.A
o India‘s climate is controlled by a number of factors which can be broadly divided into two groups
 Factors related to location and relief,
 Factors related to air pressure and winds.
o Latitude is one of the factors related to Location and Relief
 Tropic of Cancer passes through the central part of India in the east-west direction. Thus, the northern
part of India lies in the sub-tropical and temperate zone and the part lying south of the Tropic of
Cancer falls in the tropical zone. Hence statement 1 is correct.
 The tropical zone being nearer to the equator experiences high temperatures throughout the year with
small daily and annual range. Hence statement 2 is correct.
 The area north of the Tropic of Cancer being away from the equator experiences an extreme climate
with high daily and annual range of temperature.
 India lies wholly in the northern and eastern hemispheres. The mainland of India extends from 8°4'
28" N to 37°17' 53" N latitudes.
 Tropic of Cancer lies at 23°27′ N of the terrestrial Equator. It is the circle marking the latitude 23.5
degrees north, where the sun is directly overhead at noon on June 21, the beginning of summer in the
northern hemisphere. Beyond the Tropic of Cancer, Northern India does not observe overhead Sun on
the summer solstice of the northern hemisphere. Hence statement 3 is not correct.

28 www.visionias.in ©Vision IAS


Q 76.D
o Jaipur is the capital and the largest city of the Indian state of Rajasthan. It is situated at 75.8°E
longitudes.
o Bhopal is the capital city of the Indian state of Madhya Pradesh. It is situated at 77°25′E longitudes.
o Lucknow is the capital city of the Indian state of Uttar Pradesh. It is situated at 80°57′E longitudes.
o Raipur is the capital city of the Indian state of Chhattisgarh. It is situated at 81.63°E longitudes.

Q 77.D
o Earthquake waves are basically of two types - body waves and surface waves. Body waves are generated
due to the release of energy at the focus and move in all directions travelling through the body of the
earth. There are two types of body waves. They are called P and S-waves.
o P-waves move faster and are the first to arrive at the surface. These are also called ‗primary waves‘. The
P-waves are similar to sound waves. They travel through gaseous, liquid and solid materials. S-waves
arrive at the surface with some time lag. These are called secondary waves. An important fact about S-
waves is that they can travel only through solid materials. Hence statement 2 is not correct.
o Different types of earthquake waves travel in different manners. As they move or propagate, they cause
vibration in the body of the rocks through which they pass. P-waves vibrate parallel to the direction of the
wave. This exerts pressure on the material in the direction of the propagation. As a result, it creates
density differences in the material leading to stretching and squeezing of the material. The direction of
vibrations of S-waves is perpendicular to the wave direction in the vertical plane. Hence, they create
troughs and crests in the material through which they pass. Hence statement 1 is not correct.

Q 78.C
o In the course of a year, the earth‘s revolution around the Sun with its axis inclined at 66.5 degrees to the
plane of the ecliptic changes the apparent altitude of the midday Sun. The sun is vertically overhead at the
equator only on two days each year. These are usually 21st March and 21st September. These two days
are termed equinoxes. Hence statement 1 is correct.

29 www.visionias.in ©Vision IAS


o After the March equinox, the Sun appears to move north and is vertically overhead at the Tropic of
Cancer on about 21st June. This is known as the June or Summer solstice when the northern hemisphere
will have its longest day and shortest night. By about 22 December, the Sun will be overhead at the Tropic
of Capricorn. This is the Winter solstice when the southern hemisphere will have its longest day and
shortest night. Hence statement 2 is not correct.
o The tropics mark the limits of the overhead Sun, for beyond these, the Sun is never overhead at any time
of the year. Hence statement 3 is not correct.

Q 79.C
o The release of energy occurs along a fault. A fault is a sharp break in the crustal rocks. Rocks along a fault
tend to move in opposite directions. As the overlying rock strata press them, the friction locks them
together. However, their tendency to move apart at some point of time overcomes the friction. As a result,
the blocks get deformed and eventually, they slide past one another abruptly. This causes a release of
energy, and the energy waves travel in all directions.
o The point where the energy is released is called the focus of an earthquake, alternatively, it is called the
hypocentre. The energy waves travelling in different directions reach the surface. Hence statement 1 is
correct.
o The point on the surface, nearest to the focus, is called epicentre. It is the first one to experience the
waves. It is a point directly above the focus. Hence statement 2 is correct.

Q 80.A
o Alluvial soils are widespread in the northern plains and the river valleys. These soils cover about 40 per
cent of the total area of the country. Hence statement 2 is not correct.
o They are depositional soils, transported and deposited by rivers and streams. Through a narrow corridor in
Rajasthan, they extend into the plains of Gujarat. In the Peninsular region, they are found in deltas of the
east coast and in the river valleys. Hence statement 1 is correct.
o The alluvial soils vary in nature from sandy loam to clay. They are generally rich in potash but poor in
phosphorous.
o The colour of the alluvial soils varies from the light grey to ash grey. Its shades depend on the depth of the
deposition, the texture of the materials, and the time taken for attaining maturity. Alluvial soils are
intensively cultivated.
o Black soil, not alluvial soil, swell and become sticky when wet and shrink when dried. Hence statement 3
is not correct.

Q 81.B
o Statement 1 is not correct: It was Alfred Wegener, a German meteorologist who put forth a
comprehensive argument in the form of the Continental Drift theory in 1912. It was regarding the
distribution of the oceans and the continents. Arthur Holmes primary contribution was his proposed
theory that convection occurred within the Earth's mantle, which explained the push and pull of continent
plates together and apart. He also assisted scientists in oceanographic research in the 1950s, which
publicized the phenomenon known as sea floor spreading.
o Statement 2 is correct: According to Wegener, all the continents formed a single continental mass and
mega ocean surrounded the same. The supercontinent was named PANGAEA, which meant all earth. The
mega-ocean was called PANTHALASSA, meaning all water. He argued that, around 200 million years
ago, the super continent, Pangaea, began to split. Pangaea first broke into two large continental masses as
Laurasia and Gondwanaland forming the northern and southern components respectively. Subsequently,
Laurasia and Gondwanaland continued to break into various smaller continents that exist today.
o Statement 3 is not correct: Continental Drift theory suggested that the movement responsible for the
drifting of the continents was caused by pole-fleeing force and tidal force. The polar-fleeing force relates
30 www.visionias.in ©Vision IAS
to the rotation of the earth. The second force was the tidal force which is due to the attraction of the moon
and the sun that develops tides in oceanic waters. However, most scholars considered these forces to be
totally inadequate.
o Arthur Holmes is associated with Convectional Current theory. He discussed the possibility of convection
currents operating in the mantle portion. These currents are generated due to radioactive elements causing
thermal differences in the mantle portion.

Q 82.B
o Normally, temperature decreases with increase in elevation. It is called the normal lapse rate. At times, the
situation is reversed and the normal lapse rate is inverted. It is called Inversion of temperature. Inversion
is usually of short duration but quite common nonetheless.
o Statement 1 is not correct: A long winter night with clear skies and still air is an ideal situation for
inversion. The heat of the day is radiated off during the night, and by early morning hours, the earth is
cooler than the air above. Over polar areas, a temperature inversion is normal throughout the year.
o Statement 2 is correct: Surface inversion promotes stability in the lower layers of the atmosphere. Smoke
and dust particles get collected beneath the inversion layer and spread horizontally to fill the lower strata
of the atmosphere. Dense fogs in mornings are common occurrences, especially during the winter
season. This inversion commonly lasts for a few hours until the sun comes up and beings to warm the
earth.
o Statement 3 is not correct: The inversion takes place in hills and mountains due to air drainage.
Cold air at the hills and mountains, produced during night, flows under the influence of gravity. Being
heavy and dense, the cold air acts almost like water and moves down the slope to pile up deeply in
pockets and valley bottoms with warm air above. This is called air drainage. It protects plants from frost
damages.

Q 83.C
o The atmosphere is composed of gases, water vapour and dust particles. The proportion of gases changes in
the higher layers of the atmosphere in such a way that oxygen will be almost in negligible quantity at
the height of 120 km. Similarly, carbon dioxide and water vapour are found only up to 90 km from the
surface of the earth. Ozone is another important component of the atmosphere found between 10 and 50
km above the earth‘s surface and acts as a filter and absorbs the ultra-violet rays radiating from the sun
and prevents them from reaching the surface of the earth. Hence statement 1 is correct.

o An inert gas is a gas that has extremely low reactivity with other substances. The noble gases—helium,
argon, neon, xenon, krypton, radon are examples of inert gases. Hence statement 2 is correct and
statement 3 is not correct.

Q 84.C
o Jelap La is on Tibet-Bhutan border.
o Bomdi La is in Arunachal Pradesh.
o Bum La is in india-Bhutan border.
o Hence option (c) is correct.

31 www.visionias.in ©Vision IAS


Q 85.B
o The actual amount of water vapour present in the air, which is expressed in grams per cubic metre, is
called the absolute humidity. But more important from the point of view of weather studies is the relative
humidity. This is the ratio between the actual amount of water vapour and the total amount the air can
hold at a given temperature, expressed as a percentage.
o Warm air can hold more water vapour than cold air, so if it contains only half the amount it could carry,
the relative humidity is 50 per cent. In the equatorial regions, over 80 per cent is common in the morning,
'which means the air contains four-fifths as much water vapour as it can carry. When the relative
humidity reaches 100 per cent, the air is completely saturated. The air temperature is said to be at
dew-point. Further cooling will condense the water vapour into clouds or rain. It is thus clear that
when relative humidity is high the air is moist, as in the equatorial regions; when it is low, the air is dry as
in the deserts. For clouds to form and rain to start, the air does have to reach 100% relative humidity, but
only where the clouds are forming or where the rain is coming from.

Q 86.C
o Eravikulam National Park is located along the Western Ghats in the Idukki district of Kerala in India.
It is the first national park in Kerala. Eravikulam National Park is a UNESCO World Heritage Site
o The main body of the park consists of a high rolling hill plateau with a base elevation of about 2,000 m.
The terrain consists of high altitude grasslands interspersed with sholas. Anamudi, (2,695 meters), the
highest peak in India south of the Himalayas is inside this park. Many perennial streams criss-cross the
park. They merge to form tributaries of the Periyar river in the west and of the Cauvery River in the east.

32 www.visionias.in ©Vision IAS


Q 87.B
Features of Warm Temperate Eastern Margin Climate (China Type):
o Influenced by the on-shore Trade Winds all the year-round, without any monsoon variations.
o Fairly uniform distribution of rainfall throughout the year.
 There is rain every month, except in the interior of central China, where there is a distinct dry season.
Rain comes either from convectional sources or as orographic rain in summer, or from depressions in
prolonged showers in winter.
o It is sometimes referred to as the Gulf type or Natal type of climate.
This type of climate is found on the eastern margins of continents in warm temperate latitudes, just outside the
tropics as seen clearly in the below figure:

Q 88.D
o Altitude is one of the factors related to location and relief which determines the climate of a region.
Altitude affects various atmospheric parameters in the following manner:
 Temperature decreases with height. The temperature in the troposphere usually decreases with
height at the average lapse rate of 6.5 °C per kilometer.
 The density of air decreases with height. There are two reasons:
 at higher altitudes, there is less air pushing down from above, and
 gravity is weaker farther from Earth's center. So at higher altitudes, air molecules can spread out
more, and air density decreases
o Due to thin air i.e lower density, places in the mountains are cooler than places on the plains. For
example, Agra and Darjiling are located on the same latitude, but the temperature of January in Agra is
16°C whereas it is only 4°C in Darjiling.
o Pressure decreases with increasing altitude. The pressure at any level in the atmosphere may be
interpreted as the total weight of the air above a unit area at any elevation. At higher elevations, there are
fewer air molecules above a given surface than a similar surface at lower levels.

Q 89.A
o Glaciers
 Masses of ice moving as sheets over the land (continental glacier or piedmont glacier if a vast sheet of
ice is spread over the plains at the foot of mountains) or as linear flows down the slopes of mountains
in broad trough-like valleys (mountain and valley glaciers) are calledglaciers.
 The movement of glaciers is slow, unlike water flow. The movement could be a few centimetres to a
few metres a day or even less or more. Glaciers move basically because of the force of gravity.
o Erosional lanforms
 Cirques
 These are the most common of landforms in glaciated mountains. The cirques quite often are
found at the heads of glacial valleys. The accumulated ice cuts these cirques while moving down
the mountain tops. They are deep, long and wide troughs or basins with very steep concave to
vertically dropping high walls at its head as well as sides.
 Horns and Serrated Ridges
33 www.visionias.in ©Vision IAS
 Horns form through head ward erosion of the cirque walls. If three or more radiating glaciers cut
headward until their cirques meet, high, sharp pointed and steep sided peaks called horns form.
 Glaciated Valleys/Troughs
 They are trough-like and U-shaped with broad floors and relatively smooth, and steep sides. The
valleys may contain littered debris or debris shaped as moraines with swampy appearance. There
may be lakes gouged out of rocky floor or formed by debris within the valleys.
o Depositional landforms
 Moraines
 They are long ridges of deposits of glacial till. Terminal moraines are long ridges of debris
deposited at the end (toe) of the glaciers. Lateral moraines form along the sides parallel to the
glacial valleys.
 Eskers
 When glaciers melt in summer, the water flows on the surface of the ice or seeps down along the
margins or even moves through holes in the ice. These waters accumulate beneath the glacier and
flow like streams in a channel beneath the ice. Such streams flow over the ground (not in a valley
cut in the ground) with ice forming its banks. Very coarse materials like boulders and blocks
along with some minor fractions of rock debris carried into this stream settle in the valley of ice
beneath the glacier and after the ice melts can be found as a sinuous ridge called esker.
 Outwash Plains
 The plains at the foot of the glacial mountains or beyond the limits of continental ice sheets are
covered with glacio-fluvial deposits in the form of broad flat alluvial fans which may join to form
outwash plains of gravel, silt, sand and clay.
 Drumlins
 They are smooth oval shaped ridge-like features composed mainly of glacial till with some
masses of gravel and sand. The long axes of drumlins are parallel to the direction of ice
movement. They may measure up to 1 km in length and 30 m or so in height. One end of the
drumlins facing the glacier called the stoss end is blunter and steeper than the other end called tail.

Q 90.A
o A river stream forms a series of landforms on its course. The formation of landforms by a river stream can
be divided into three phases -:
 Youth
 Streams are few during this stage with poor integration and flow over original slopes showing
shallow V-shaped valleys with no floodplains or with very narrow floodplains along trunk
streams. Streams divides are broad and flat with marshes, swamp and lakes. Meanders if present
develop over these broad upland surfaces. These meanders may eventually entrench themselves
into the uplands. Waterfalls and rapids may exist where local hard rock bodies are exposed.
 Mature
 During this stage streams are plenty with good integration. The valleys are still V-shaped but
deep; trunk streams are broad enough to have wider floodplains within which streams may flow in
meanders confined within the valley. The flat and broad inter stream areas and swamps and
marshes of youth disappear and the stream divides turn sharp. Waterfalls and rapids disappear.
 Old
 Smaller tributaries during old age are few with gentle gradients. Streams meander freely over vast
floodplains showing natural levees, oxbow lakes, etc. Divides are broad and flat with lakes,
swamps and marshes. Most of the landscape is at or slightly above sea level.
o Hence, option (a) is the correct answer.

Q 91.C
o All processes that move, elevate or build up portions of the earth‘s crust come under diastrophism. They
include:
 orogenic processes involving mountain building through severe folding and affecting long and narrow
belts of the earth‘s crust;
 epeirogeny involves uplift or warping of large parts of the earth‘s crust;
 earthquakes involving local relatively minor movements;
 plate tectonics involving horizontal movements of crustal plates.
o Orogeny is a mountain building process whereas epeirogeny is a continental building process. In the
process of orogeny, the crust is severely deformed into folds. Due to epeirogeny, there may be simple
deformation. Hence statement 1 is not correct.
34 www.visionias.in ©Vision IAS
o Orogeny and Epeirogeny both are caused by endogenic forces. Thus, the forces required for them
emanates from within the earth. It is mostly generated by radioactivity, rotational and tidal friction and
primordial heat from the origin of the earth. Hence statement 2 is not correct.

Q 92.A
o Soil is a result of decay and medium for growth. It is a changing and a developing body. It may be
alternatively cold and warm or dry and moist. There are five basic factors control the formation of soils:
(i) parent material; (ii) topography; (iii) climate; (iv) biological activity; (v) time.
o Biological activity is slowed or stopped if the soil becomes too cold or too dry.
o Intensity of bacterial activity shows up differences between soils of cold and warm climates.
o Humus accumulates in cold climates as bacterial growth is slow. With undecomposed organic matter
because of low bacterial activity, layers of peat develop in sub-arctic and tundra climates. Hence
statement 1 is correct.
o In humid tropical and equatorial climates, bacterial growth and action is intense and dead
vegetation is rapidly oxidised leaving very low humus content in the soil. Hence statement 2 is not
correct. Further, bacteria and other soil organisms take gaseous nitrogen from the air and convert it into a
chemical form that can be used by plants. This process is known as nitrogen fixation.
o The vegetative cover and organisms that occupy the parent materials from the beginning and also at later
stages help in adding organic matter, moisture retention, nitrogen etc. Dead plants provide humus, the
finely divided organic matter of the soil. Some organic acids which form during humification aid in
decomposing the minerals of the soil parent materials.

Q 93.D
o The continental shelf is a gently sloping and relatively flat extension of a continent that is covered by the
oceans. Seaward, the shelf ends abruptly at the shelf break, the boundary that separates the shelf from the
continental slope.
o Continental shelves contain valuable resources, such as oil and gas and minerals. Oil and gas are formed
from organic material that accumulates on the continental shelf. Over time the material is buried and
transformed into oil and gas by heat and pressure.
o A placer deposit is an accumulation of valuable minerals formed by gravity separation from a specific
source rock during sedimentary processes. Minerals come from rocks on land and are carried to the ocean
by rivers. The minerals are deposited in river channels and beaches. Examples of important minerals on
the shelf are diamonds, chromite (chromium ore), limenite (titanium ore), magnetite (iron ore),
platinum, and gold.
o Continental shelves are home to the largest concentration of bottom-dwelling marine life, and they are
the sites of major fishing grounds.

Q 94.B
o Tropical cyclones originate and intensify over warm tropical oceans. The conditions favourable for the
formation and intensification of tropical storms are:
 Large sea surface with a temperature higher than 27° C; Hence statement 1 is not correct.
 Presence of the Coriolis force;
 Small variations in the vertical wind speed;
 A pre-existing weak low-pressure area or low-level-cyclonic circulation;
 Upper divergence above the sea level system.
o The cyclones, which cross 20°N latitude generally, recurve and they are more destructive (cyclones
do not form near the equator due to the absence of Coriolis force). Hence statement 3 is not correct.
o A mature tropical cyclone is characterized by the strong spirally circulating wind around the centre, called
the eye. The diameter of the circulating system can vary between 150 and 250 km.
o The eye is a region of calm with subsiding air. Around the eye is the eyewall, where there is a strong
spiralling ascent of air to a greater height reaching the tropopause. The wind reaches maximum velocity in
this region, reaching as high as 250 km per hour. Torrential rain occurs here. Hence statement 2 is
correct.
o From the eyewall, rain bands may radiate and trains of cumulus and cumulonimbus clouds may drift into
the outer region. The diameter of the storm over the Bay of Bengal, the Arabian Sea and Indian ocean is
between 600 and 1200 km.
o The system moves slowly about 300 - 500 km per day. The cyclone creates storm surges and they
inundate the coastal low lands. The storm peters out on the land.

35 www.visionias.in ©Vision IAS


Q 95.D
o The rotation of the earth about its axis affects the direction of the wind. This force is called the Coriolis
force after the French physicist who described it in 1844. It deflects the wind in the right direction in the
northern hemisphere and to the left in the southern hemisphere. The deflection is more when the wind
velocity is high. The Coriolis force is directly proportional to the angle of latitude. It is maximum at
the poles and is absent at the equator. Hence, statement 3 is correct.
o The Coriolis force acts perpendicular to the pressure gradient force. The pressure gradient force is
perpendicular to an isobar (a line connecting points having the same atmospheric pressure). The higher the
pressure gradient force, the more is the velocity of the wind and the larger is the deflection in the direction
of the wind. As a result of these two forces operating perpendicular to each other, in the low-pressure
areas the wind blows around it. At the equator, the Coriolis force is zero and the wind blows
perpendicular to the isobars. The low pressure gets filled instead of getting intensified. That is the
reason why tropical cyclones are not formed near the equator. Hence, statement 1 is correct.
o The velocity and direction of the wind are the net results of the wind generating forces. The winds in the
upper atmosphere, 2 - 3 km above the surface, are free from frictional effect of the surface and are
controlled by the pressure gradient and the Coriolis force. When isobars are straight and when there is no
friction, the pressure gradient force is balanced by the Coriolis force and the resultant wind blows
parallel to the isobar. This wind is known as the geostrophic wind. Hence, statement 2 is correct.

Q 96.C
o Classification of Natural Disasters:
 Atmospheric:
 Blizzards
 Thunderstorms
 Lightning
 Tornadoes
 Tropical Cyclone
 Drought
 Hailstorm
 Frost, Heat Wave or Loo.Cold Waves, etc.
 Terrestrial:
 Earthquakes
 Volcanic Eruptions
 Landslides
 Avalanches
 Subsidence
 Soil Erosion
 Aquatic:
 Floods
 Tidal Waves
 Ocean Currents
 Storm Surge
 Tsunami
 Biological:
 Plants and Animals as colonisers (Locusts, etc.).
 Insects infestation— fungal, bacterial and viral diseases such as bird flu, dengue, etc.

Q 97.B
o In India, the mangrove forests are highly developed in the Andaman and Nicobar Islands and the
Sunderbans of West Bengal. Other areas of significance are the Mahanadi, the Godavari and the Krishna
deltas. As per the Indian State of Forest Report 2017, mangrove cover in states are as follows :
o Andhra Pradesh - 404 sq km
o Gujarat - 1140 sq km
o Maharashtra - 304 sq km
o Odisha - 243 sq km
o West Bengal - 2114 sq km
o Andaman & Nicobar Islands - 617 sq km

36 www.visionias.in ©Vision IAS


Q 98.B
o The Ganges River, also called Ganga, is a river located in northern India that flows toward the border with
Bangladesh. It is the longest river in India and flows for around 1,569 miles (2,525km) from the
Himalayan Mountains to the Bay of Bengal.
o The Ganges passes through the states of Uttarakhand, Uttar Pradesh, Bihar, Jharkhand, and West
Bengal.The Ganga enters the plains at Haridwar. From here, it flows first to the south, then to the south-
east and east before splitting into two distributaries, namely the Bhagirathi and the Hugli. The river has a
length of 2,525 km. It is shared by Uttaranchal (110 km) and Uttar Pradesh (1,450 km), Bihar (445 km)
and West Bengal (520 km) and Sahibganj town of Jharkhand is located on the bank of River Ganges.
The Ganga basin covers about 8.6 lakh sq. km area in India alone.

Q 99.D
o The speed of the wave in the ocean depends upon the depth of water. Water waves travel slower in
shallower water. The deeper the water the faster the waves can travel, in fact in the ocean waves speeds
can sometimes approach hundreds of miles per hour. As a result of this, the impact of the tsunami is less
over the ocean and more near the coast where they cause large-scale devastations. Therefore, a ship at sea
is not much affected by the tsunami and it is difficult to detect a tsunami in the deeper parts of the
sea. Hence option 1 is not correct.
o It is so because over deep water the tsunami has very long wave-length and limited wave-height. Thus, a
tsunami wave raises the ship only a meter or two and each rise and fall takes several minutes.
o As opposed to this, when a tsunami enters shallow water, its wave-length gets reduced and the
period remains unchanged, which increases the wave height. Sometimes, this height can be up to 15m
or more, which causes large-scale destructions along the shores. Thus, these are also called Shallow Water
Waves. Hence option 2 is not correct and option 3 is correct.

Q 100.D
o Frequent inundation of agricultural land and human settlement have serious consequences on the national
economy and society.
o Floods do not only destroy valuable crops every year but these also damage physical infrastructure such as
roads, rails, bridges and human settlements. Millions of people are rendered homeless and are also washed
down along with their cattle in the floods. Hence option 1 is correct.
o Spread of diseases like cholera, gastro-enteritis, hepatitis and other water-borne diseases spread in the
flood-affected areas. Hence option 2 is correct.
o However, floods also make a few positive contributions. Every year, floods deposit fertile silt over
agricultural fields which is good for the crops. Majuli (Assam), the largest riverine island in the world, is
the best example of good paddy crops after the annual floods in Brahmaputra. Hence option 3 is correct.

A initiative to provide watermarks and bookmarks free pdfs to you.


Share and Subscribe our telegram channel
@visionpt3652019

https://t.me/visionpt3652019

Copyright © by Vision IAS


All rights are reserved. No part of this document may be reproduced, stored in a retrieval system or transmitted
in any form or by any means, electronic, mechanical, photocopying, recording or otherwise, without prior
permission of Vision IAS

37 www.visionias.in ©Vision IAS

You might also like